If the table is auctioned on a date that is later than both the date on which the mirror is auctioned and the date on...

Jasmin1 on July 19, 2021

What about h-s-m/v-m/v-t-l

If we used h-s-m/v-m/v-t-l H would be before T and still meet the m&v-T. So couldn't A be possible?

Reply
Create a free account to read and take part in forum discussions.

Already have an account? log in

Emil-Kunkin on January 26 at 08:38PM

This violates that last rule, the t must be before one of m or v